The altitude (or height) of a triangle is increasing at a rate of 2.5cm/min while the area of the triangle is increasing at a rate of 3cm2/min. At what rate (in cm/min) is the base of the triangle changing when the altitude is 12cm and the area is 84cm2 Round your answer to three decimal places.

Answers

Answer 1

The base is decreasing at 1.917 cm/min when altitude is 12cm and  area=84cm².

Let A be the area of the triangle, h be the height of the triangle, and b be the base of the triangle.

Then the formula for the area of a triangle is:

A = (1/2)bh

We are given that dh/dt = 2.5 cm/min (the height is increasing at a rate of 2.5cm/min), and dA/dt = 3 cm²/min (the area is increasing at a rate of 3cm²/min).

We want to find db/dt, the rate of change of the base of the triangle when h = 12 cm and A = 84 cm².

To solve this problem, we need to use the chain rule of differentiation.

We start by differentiating both sides of the formula for the area of a triangle with respect to time t:

dA/dt = (1/2) d/dt (bh)

Next, we can use the product rule of differentiation to find d/dt (bh):

d/dt (bh) = b dh/dt + h db/dt

Substituting this into the previous equation gives:

dA/dt = (1/2) [ b dh/dt + h db/dt ]

Now we can substitute the given values of dh/dt and dA/dt, as well as h = 12 cm and A = 84 cm².

To find db/dt:

3 cm²/min = (1/2) [ b (2.5 cm/min) + 12 cm db/dt ]

Simplifying this expression gives:

6 cm²/min = 2.5 b cm²/min + 12 cm db/dt

Substituting A = 84 cm² and h = 12 cm into the formula for the area of a triangle gives:

84 cm² = (1/2) b (12 cm)

Simplifying this expression gives:

b = 14 cm

Now we can substitute b = 14 cm into the previous equation to find db/dt:

6 cm²/min = 2.5 (14 cm) cm²/min + 12 cm db/dt

Simplifying this expression gives:

db/dt = (6 cm²/min - 35 cm²/min) / (12 cm)

db/dt = -1.917 cm/min (rounded to three decimal places)

Therefore, the base of the triangle is decreasing at a rate of 1.917 cm/min when the height is 12 cm and the area is 84 cm².

To practice more questions about chain rule:

https://brainly.com/question/30895266

#SPJ11


Related Questions

The width of the large size is 9.9 cm and its height is 19.8 cm.
The width of the small size bottle is 4.5 cm.
hcm
h =
4.5 cm
Calculate the height of the small bottle.
19.8 cm
9.9 cm
+
cm

Answers

Answer and Explanation:

The height of the small bottle can be calculated using the ratio of the width of the large and small bottles.

Ratio of width = Large bottle width / Small bottle width

Ratio of width = 9.9 cm / 4.5 cm

Ratio of width = 2.2

Therefore, the height of the small bottle can be calculated by multiplying the ratio of width with the height of the large bottle.

Height of small bottle = Ratio of width x Height of large bottle

Height of small bottle = 2.2 x 19.8 cm

Height of small bottle = 43.56 cm

7. A rectangular prism has a volume of 135ft^3. The width of the rectangular prism is (2x+10)ft. The height of the rectangular prism is 5 times it's width. Write a expression that gives the length of the rectangular prism in feet?



A. 4(x+5)/27 B. 27/4(x+5)



C. (2x^2+100)/27. D. 27/(2x^2+100)

Answers

The expression that gives the length of the rectangular prism in feet is option D: 27/(2x^2+100).

What is the expression that gives the length of the rectangular prism in feet?

The volume of a rectangular prism is given by the formula V = lwh, where l is the length, w is the width, and h is the height.

We are given that the volume of the rectangular prism is 135ft^3, and the width is (2x+10)ft. Also, the height is 5 times the width, so h = 5w.

Substituting these values in the formula for the volume, we get:

135 = l(2x+10)(5w)

Dividing both sides by (2x+10)(5w), we get:

l = 135 / (2x+10)(5w)

l = 135 / [10(x+5)w]

Now we can substitute h = 5w:

l = 135 / [10(x+5)h/5]

l = 135 / [2(x+5)h]

l = 135 / [2(x+5)(5w)]

l = 135 / [10(x+5)^2]

Simplifying the expression, we get:

l = 27 / (2(x+5)^2)

Therefore, the expression that gives the length of the rectangular prism in feet is option D: 27/(2x^2+100).

Learn more about rectangular prism in feet

brainly.com/question/30956442

#SPJ11

The length of a rectangle is 6 ft longer than its width. if the perimeter of the rectangle is 64 ft, find its length and width

Answers

The length of the rectangle is 19 feet and its width is 13 feet.

Let's denote the width of the rectangle by w. Then, according to the problem statement, the length of the rectangle is 6 feet longer, which means it is equal to w + 6.

The perimeter of a rectangle is given by the formula:

perimeter = 2 × length + 2 × width

Substituting the expressions for length and width that we have just found, we get:

64 = 2 × (w + 6) + 2w

Simplifying the right-hand side:

64 = 2w + 12 + 2w

64 = 4w + 12

52 = 4w

w = 13

So the width of the rectangle is 13 feet. Using the expression for the length we found earlier, the length is:

length = w + 6 = 13 + 6 = 19

Therefore, the length of the rectangle is 19 feet and its width is 13 feet.

To know more about rectangle, refer to the link below:

https://brainly.com/question/28711757#

#SPJ11

In the last 215 days, builders have completed 700 m2 of the alligator habitat that will eventually be 1,200 m2. How much longer will it take to complete the alligator habitat?

Answers

In the last 215 days, builders have completed 700 m2 of the alligator habitat that will eventually be 1,200 m2.
It will take approximately 153 days to complete the remaining part of the alligator habitat.

Determine how much longer it will take to complete the alligator habitat, first, we need to find the rate at which the builders are working.
Calculate the work rate
The builders have completed 700 m2 of the 1,200 m2 alligator habitat in 215 days.
Work rate = (completed work) / (number of days)
Work rate = 700 m2 / 215 days = 3.26 m2/day (approximately)
Calculate the remaining work
The total area of the alligator habitat is 1,200 m2, and 700 m2 has been completed.
Remaining work = Total area - Completed work
Remaining work = 1,200 m2 - 700 m2 = 500 m2
Calculate the time to complete the remaining work
Time to complete = (remaining work) / (work rate)
Time to complete = 500 m2 / 3.26 m2/day ≈ 153.37 days
It will take approximately 153 days to complete the remaining part of the alligator habitat.

Read more about alligator habitat.

https://brainly.com/question/15823089

#SPJ11

It will take approximately 394 more days to complete the alligator habitat.

We can start by finding the proportion of the habitat that has already been completed:

proportion completed = 700 m^2 / 1200 m^2 = 0.5833

This means that there is still 1 - 0.5833 = 0.4167 (or 41.67%) of the habitat left to complete.

Next, we can use a proportion to find out how long it will take to complete the remaining 41.67% of the habitat:

215 days / 0.5833 = x days / 0.4167

Solving for x, we get:

x = 215 days * 0.4167 / 0.5833 ≈ 153 days

Therefore, the total time it will take to complete the alligator habitat is approximately 215 + 153 = 368 days, or about 394 more days from the start.

For more questions like Alligator click the link below:

https://brainly.com/question/154098

#SPJ11

1 pts How much bubble wrap is needed to cover a cylindrical vase that is 16 inches tall with a diameter of 6 inches?​

Answers

415 square inches of bubble wrap to cover the cylindrical vase that is 16 inches tall with a diameter of 6 inches.

To calculate how much bubble wrap is needed to cover the cylindrical vase, you will need to find the circumference and height of the vase.

First, calculate the circumference of the vase using the diameter of 6 inches:
Circumference = π x diameter
Circumference = 3.14 x 6
Circumference = 18.84 inches

Next, calculate the height of the vase which is given as 16 inches.

To find the surface area of the vase, you will need to multiply the circumference by the height and add the area of the circular bases. The formula for the surface area of a cylinder is:

Surface area = 2πr² + 2πrh
where r is the radius and h is the height.

Since the vase has circular bases, we can find the area of each base by using the formula:
Area of circle = πr²

Now, let's find the radius of the vase:
[tex]Radius = \frac{diameter}{2}[/tex]
[tex]Radius = \frac{6}{2}[/tex]
Radius = 3 inches

So, the area of each base is:

Area of base = π x (radius)²
Area of base = π x 3²
Area of base = 28.27 square inches

The total area of the two bases is 2 x 28.27 = 56.54 square inches.

Now, let's find the surface area of the cylinder:

Surface area = 2πr² + 2πrh
Surface area = 2 x π x 3² + 2 x π x 3 x 16
Surface area = 113.1 + 301.44
Surface area = 414.54 square inches

Therefore, you would need approximately 415 square inches of bubble wrap to cover the cylindrical vase that is 16 inches tall with a diameter of 6 inches.

To know more about "surface area of a cylinder" refer here:

https://brainly.com/question/28575608#

#SPJ11

Sergey is solving 5x2 + 20x – 7 = 0. Which steps could he use to solve the quadratic equation by completing the square? Select three options

Answers

The quadratic equation where the squares had been completed is:

(x + 2)² = 27/5

How to complete squares?

Remember the perfect square trinomial:

(a + b)² = a² + 2ab + b²

now we have the quadratic equation:

5x² + 20x - 7 = 0

If we divide it all by 5, we will get.

x² + 4x - 7/5 = 0

Now we can rewrite this as:

(x² + 2*2*x )  - 7/5 = 0

Now we need to add 2² in both sides, we will get:

(x² + 2*2x + 2²) - 7/5 = 2²

(x + 2)² = 4 + 7/5

(x + 2)² = 27/5

There the square is completed.

Learn more about completing squares at:

https://brainly.com/question/10449635

#SPJ1

900,000=x+y+z
79,750=0. 08x+0. 09y+0. 01z
2x=z

Answers

Answer:

since 2x = z

replace z with 2x

900000 = x+y+z

900000 = x+y+2x

900000 = 3x+y - eqn (1)

79750= 0.08x +0.09y+0.01z

79750 = 0.08x +0.09y+0.01(2x)

79750 = 0.08x+0.09y+0.02x

79750 = 0.10x +0.09y - eqn(2)

from eqn(1)

900000 = 3x + y

y = 900000-3x - eqn(3)

substitute eqn(3) in eqn(2)

79750 = 0.1x +0.09y

79750=0.1x + 0.09(900000-3x)

79750=0.1x+ 81000 - 0.27x

collect like terms

79750 -81000 = 0.1x-0.27x

-1250 = -0.17x

to find x divide both sides by -0.17

x = -1250/-0.17 ~= 7353

since 2x = z

2*7353 = 14706

in eqn(3)

y = 900000-3x

y= 900000-3(7353)

y = 900000-22059

y = 877941

x =7353,y= 877941,z=14706

After a windstorm, a leaning pole makes a 75° angle with the road surface. the pole casts a 15-foot shadow when the sun is at a 45° angle of elevation. about how long is the pole?

Answers

The pole is approximately 3.86 feet tall.

What is the length of a leaning pole that makes a 75° angle with the road surface, if it casts a 15-foot shadow when the sun is at a 45° angle of elevation?

Let's denote the height of the pole as "x" (in feet). From the problem, we know that the pole makes a 75° angle with the road surface, which means that the angle between the pole and the vertical is 90° - 75° = 15°.

Now, we can use the tangent function to find the height of the pole:

tan(15°) = x/15

Multiplying both sides by 15, we get:

x = 15 tan(15°) ≈ 3.86 feet

Learn more abut length

brainly.com/question/9842733

#SPJ11

Question 1(Multiple Choice Worth 4 points)


A funnel is shaped like a cone and is 4. 5 inches high and has a diameter of 6 inches. What is the volume of the funnel? Use 3. 14 for pi. Round your answer to the nearest hundredth. 10. 60 in3

42. 39 in3

63. 61 in3

169. 64 in3

Answers

The volume of the funnel is approximately 42.39 in³. The correct answer is option 2.

To calculate the volume of the funnel, which is shaped like a cone, we need to use the formula for the volume of a cone: V = (1/3)πr²h.

Given:
Height (h) = 4.5 inches
Diameter = 6 inches
Radius (r) = Diameter / 2 = 6 / 2 = 3 inches
Pi (π) ≈ 3.14

Now, plug the values into the formula:

V = (1/3) × 3.14 × 3² × 4.5
V ≈ (1/3) × 3.14 × 9 × 4.5
V ≈ 3.14 × 3 × 4.5
V ≈ 42.39 in³

So, the volume of the funnel is approximately 42.39 in³. The correct answer is option 2.

To learn more about volume, refer below:

https://brainly.com/question/1578538

#SPJ11

(n+3)!/(n+1)! please help immediately

Answers

Answer:

(n + 3)(n + 2) or n² + 5n + 6

------------------

The factorial of a non-negative integer n, denoted by n!, is the product of all positive integers less than or equal to n:

n! = n × (n - 2) × (n - 3) × ... × 2 × 1

As per above mentioned definition we see that:

(n + 3)! = (n + 3) × (n + 2) × (n + 1)!

Hence the quotient of (n + 3)! and (n + 1)! is:

(n + 3)(n + 2) or n² + 5n + 6

Casho went shopping for a new pair of sneakers because of a sale. The price on the tag was $25, but Casho paid $22. 50 before tax. Find the percent discount

Answers

The percent discount on the sneakers is 10%

Casho paid $22.50 before tax, despite the item's $25 tag price. The discount is the difference between the original price and the sale price, which is $25 - $22.50 = $2.50.

The discount is the difference between the original price and the discounted price, expressed as a percentage of the original price.

To find the percent discount, we divide the discount by the original price and multiply by 100:

Percent discount = (discount / tag price) x 100

Percent discount = ($2.50 / $25) x 100

Percent discount = 0.1 x 100

Percent discount = 10%

Therefore, the percent discount on the sneakers is 10%

Learn more about Discount here

https://brainly.com/question/3541148

#SPJ4

A portion of an electrical circuit is displayed next. the switches operate independently of each other, and the probability that each relay closes when the switch is thrown is displayed by the switch. what is the probability that current will flow from s to t when the switch is thrown

Answers

If you provide me with a specific circuit diagram and the relevant details, I would be happy to help you determine the probability of current flowing from s to t when the switch is thrown.

What is the probability of current flowing from s to t when the switch is thrown?

I apologize, but it seems that the circuit diagram you mentioned is not displayed here. Without the circuit diagram, it is not possible for me to provide a specific answer to your question.

However, in general, the probability of current flowing from s to t in an electrical circuit depends on several factors such as the voltage level, the resistance of the circuit components, and the state of the switches. If the switches are all closed, then the probability of current flowing from s to t will depend on the overall resistance of the circuit.

If you provide me with a specific circuit diagram and the relevant details, I would be happy to help you determine the probability of current flowing from s to t when the switch is thrown.

Learn more about specific circuit

brainly.com/question/30888688

#SPJ11

do you believe your children will have a higher standard of living than you have? this question was asked of a national sample of american adults with children in time/cnn poll. sixty-three percent answered in the affirmatve, with a margin of error or plys or minus 3%. assume that the true percentage of all american adults who beleive their children with have a hgiehr standard of living is .60

Answers

True percentage of all American believes that their children have higher standard of living with confidence interval of 95% is between 60% and 66% .

CI is the confidence interval

Answered in the affirmative =  63%

p is the sample proportion =0.63

z is the critical value from the standard normal distribution at the desired confidence level

Using attached z-score table,

95% confidence level corresponds to z=1.96

n is the sample size

Use the margin of error ,

Calculate a confidence interval for percentage of American adults who believe their children will have a higher standard of living.

A margin of error of plus or minus 3% means ,

95% confident that the true percentage falls within 3% of the sample percentage.

Using the formula for a confidence interval for a population proportion,

CI = p ± z×√(p(1-p)/n)

Plugging in the values, we get,

⇒ CI = 0.63 ± 1.96√(0.63(1-0.63)/n)

Solving for n, we get,

n = (1.96/0.03)^2 × 0.63(1-0.63)

⇒ n = 994.87

Rounding up to the nearest whole number, sample size of at least 995.

⇒ CI = 0.63 ± 1.96√(0.63(1-0.63)/995)

⇒CI = 0.63 ± 0.02999

95% confidence interval for the true percentage is,

⇒CI = 0.63 ± 0.03

⇒CI = (0.60, 0.66)

Therefore, 95% confidence interval that between 60% and 66% of all American adults with children believe that their children will have a higher standard of living.

Learn more about confidence interval here

brainly.com/question/16409874

#SPJ4

12
Find the first and second derivatives. S = 15 + 344 - 1 15 S' = S'' =

Answers

The first derivative of S is S' = 1/15.
The second derivative of S is S'' = 0.



To find the first derivative (S'):

Starting with the given equation S = 15 + 344 - 1 15, we can simplify it to S = 344 + 15.

We can take the derivative of each term separately since they are added together.

The derivative of a constant (15 and 344) is always 0, so we only need to take the derivative of 1/15.

S' = d/dx (344 + 15)

= d/dx (359)

= 0 + 0 + (d/dx (1/15))

= 1/15

Therefore, the first derivative of S is S' = 1/15.

To find the second derivative (S''):

We need to take the derivative of the first derivative (S').

Since the derivative of a constant is always 0,

we only need to take the derivative of 1/15.

S'' = d/dx (S')

= d/dx (1/15)

= 0

Therefore, the second derivative of S is S'' = 0.

To know more about Derivatives:

https://brainly.com/question/25324584

#SPJ11

Roxie plans on purchasing a new desktop computer for $1250. Which loan description would result in the smallest monthly payment when she pays the loan back?



12 months at 6. 25% annual simple interest rate



18 months at 6. 75% annual simple interest rate



24 months at 6. 5% annual simple interest rate



30 months at 6. 00% annual simple interest rate

Answers

The loan with the smallest monthly payment is the 30-month loan at 6% annual simple interest rate, with a monthly payment of $45.83.

To determine the loan with the smallest monthly payment, we need to calculate the monthly payment for each loan option and compare them.

We can use the formula for monthly payment on a simple interest loan:

monthly payment = (principal + (principal * interest rate * time)) / total number of payments

where:

principal is the amount borrowed (in this case, $1250)interest rate is the annual simple interest rate divided by 12 to get the monthly ratetime is the length of the loan in months

We can compute the monthly payments for each loan choice using this formula:

1. 12 Monthly interest rate = 0.0625/12 = 0.00521, monthly payment = (1250 + (1250 * 0.00521 * 12)) / 12 = $107.35

2. 18 months at 6.75%: monthly interest rate = 0.0675/12 = 0.00563, monthly payment = (1250 + (1250 * 0.00563 * 18)) / 18 = $81.96

3. 24 months at 6.5%: monthly interest rate = 0.065/12 = 0.00542, monthly payment = (1250 + (1250 * 0.00542 * 24)) / 24 = $66.14

4. 30 months at 6%: monthly interest rate = 0.06/12 = 0.005, monthly payment = (1250 + (1250 * 0.005 * 30)) / 30 = $45.83

Based on these calculations, the loan with the smallest monthly payment is the 30-month loan at 6% annual simple interest rate, with a monthly payment of $45.83.

Read more on simple interest on:

brainly.com/question/24429956

#SPJ4

In a survey, the planning value for the population proportion is p* = 0.25. How large a sample should be taken to provide a 95% confidence interval with a margin of error of 0.02? Round your answer up to the next whole number. How large a sample should be selected to provide a 95% confidence interval with a margin of error of 2? Assume that the population standard deviation is 30. Round your answer to next whole number.

Answers

The Sample size that is necessary for the selection is =7203

How to solve

Given that,

[tex]\hat p= 0.25[/tex]

[tex]1 - \hat p = 1 - 0.25 = 0.75[/tex]

margin of error = E = 0.01

At 95% confidence level the z is ,

\alpha = 1 - 95% = 1 - 0.95 = 0.05

[tex]\alpha / 2 = 0.05 / 2 = 0.025[/tex]

Z\alpha/2 = Z0.025 = 1.96 ( Using z table )

Sample size = n = [tex](Z\alpha/2 / E)2 * \hat p * (1 - \hat p)[/tex]

= (1.96 / 0.01)2 * 0.25 * 0.75

= 7203

Sample size =7203

In statistics, the sample size is the measure of the number of individual samples used in an experiment.

The size of the sample holds significant importance in any empirical study that aims to draw conclusions about a larger population based on a smaller sample.

Read more about sample size here:

https://brainly.com/question/30224379

#SPJ1

Please help ASAP!!!!! In a certain Spanish class of 30 students, 11 of them play basketball and 15 of them play baseball. There are 10 students who play both sports. What is the probability that a student chosen randomly from the class plays basketball or baseball? Answer


should be a fraction in simplest form

Answers

The probability that a student chosen randomly from the class plays basketball or baseball is 8/15

Total number of students in Spanish class = 30

Student who plays basketball (A) = 11

Student who plays baseball (B) = 15

Student who plays both sports (A and B) = 10

To find a student who plays basketball or baseball (A or B)

(A or B)  = A + B -  (A and B)

(A or B) = 11 +15 -10

(A or B) = 16

P(A or B) = No. of favorable outcome/ Total no. of outcomes

P(A or B) = 16/30

In simplest form = 8/15

To know more about probability click here :

https://brainly.com/question/30034780

#SPJ4

Now that you have chosen your mode of transportation, use your choice to answer the questions that follow.





What would the cost of your transportation be if you drove:

a. 10 miles? b. 25 miles? c. 42 miles? d. 68 miles?

Make sure to list your chosen mode of transportation and then answer all parts and show your work

Answers

(a) The cost of City Bus for driving 10 miles = $3.

(b) The cost of City Bus for driving 25 miles = $7.5.

(c) The cost of City Bus for driving 42 miles = $12.6.

(d) The cost of City Bus for driving 68 miles = $20.4.

We previously choose City Bus as our mode transport since the per mile cost for City Bus is less.

Let the model for City Bus be f(x) = cx + d, where f(x) is total cost and x is number of miles.

From the table of Taxi we get, f(2) = 0.60; f(4) = 1.20; f(6) = 1.80 and f(8) = 2.40.

So, 2a + b = 0.60 and 4a + b = 1.20

(4a + b) - (2a + b) = 1.20 - 0.60

2a = 0.60

a = 0.60/2 = 0.30

Now, f(8) = 2.40

8*0.30 + b = 2.40

2.40 + b = 2.40

b = 2.40 - 2.40 = 0

So the function rule for City Bus is, f(x) = 0.3x.

(a) Total cost to drive 10 miles is,

f(10) = 0.3*10 = 3

(b) Total cost to drive 25 miles is,

f(25) = 0.3*25 = 7.5

(c) Total cost to drive 42 miles is,

f(42) = 0.3*42 = 12.6

(d) Total cost to drive 68 miles is,

f(68) = 0.3*68 = 20.4

To know more about function rule here

https://brainly.com/question/30139621

#SPJ1

Select all of the following that represent the part of the grid that is shaded.



A ten-by-ten grid has 7 columns shaded.



A.


70


100


B.


7


10


C.


70


10


D.


0. 07



E.


0. 7

Answers

A ten-by-ten grid has 7 columns shaded. All of the following that represent the part of the grid that is shaded are : The correct answer is (A) 70 and (B) 7.

The information given in the problem tells us that a ten-by-ten grid has 7 columns shaded. Since there are a total of 10 columns in the grid, this means that 7/10 of the columns are shaded.

To express this as a percentage, we can divide 7 by 10 and multiply by 100:

(7/10) x 100 = 70%

Therefore, 70 represents the percentage of columns that are shaded in the grid. Option (A) is correct.

Alternatively, we can express the same proportion as a decimal by dividing 7 by 10:

7/10 = 0.7

Therefore, 0.7 represents the proportion of columns that are shaded in the grid. Option (E) is incorrect because it shows 0.7 as a fraction instead of a decimal.

Option (B) is also correct because it correctly identifies the number of shaded columns as 7. Option (C) is incorrect because it includes both the percentage and the number of shaded columns, which is redundant. Option (D) is incorrect because it shows the proportion of shaded columns as a decimal with an extra zero.

To know more about grid, refer to the link below:

https://brainly.com/question/29774894#

#SPJ11

Pythagorean theorem
I need help with my math

Answers

The height of the flagpole is 26.0 feet.

What is height?

Height is the vertical distance between two points.

To calculate the height of the flagpole, we use the formula below.

Formula:

h = √(l²-d²)............... Equation 1

Where:

h = Height of the flagpolel = Length of the wired = Distance of the wire from the ground to the base of the pole

From the question,

Given:

l = 300 feetd = 15 feet

Substitute these values into equation 1

d = √(30²-15²)d = √675d = 26.0 feet

Learn more about height here: https://brainly.com/question/12446886

#SPJ1

Suppose that in 1682, a man bought a diamond for $32. Suppose that the man had instead put the $32 in the bank at 3% interest compounded continuously. What would that $32 have been worth in 2003? In 2003, the $32 would have been worth $ (Do not round until the final answer. Then round to the nearest dollar as needed.)

Answers

If a man bought a diamond for $32 in 1682 and the man had instead put the $32 in the bank at 3% interest compounded continuously, then the value of the diamond in 2003 would be $554,311.

The given problem is related to exponential growth. In this problem, the continuous compounding formula will be used to find the value of $32 in 2003.

The formula for continuous compounding is given by:

A = Pert   Where,

P is the principal amount,

r is the annual interest rate,

e is the Euler's number which is approximately 2.71828, and

t is the time in years.

Using the formula, we get:

A = 32e^(0.03 x 321)

A = 32e^9.63

A = 32 x 17322.23

A = $ 554311.36

Thus, $32 invested at 3% compounded continuously from 1682 to 2003 would be worth $554,311.

To know more about interest, click on the link below:

https://brainly.com/question/30393144#

#SPJ11

In a regular tiling, if there are six polygons meeting at a vertex, then the angles at the vertex are _____ degrees

Answers

In a regular tiling, if there are six polygons meeting at a vertex, then the angles at the vertex are 120 degrees.

This is because each regular polygon has interior angles that are multiples of 180 degrees divided by the number of sides. For a regular hexagon, which has six sides, each interior angle measures 120 degrees. When six regular hexagons meet at a vertex in a regular tiling, the total angle sum at the vertex is 720 degrees (6 times 120 degrees).

Since the angles must be divided equally among the six hexagons, each angle at the vertex is 120 degrees.

Learn more about interior angles:

https://brainly.com/question/24966296

#SPJ11

If one line passes through the points (-3,8) & (1,9), and a perpendicular line passes through the point (-2,4), what is another point that would lie on the 2nd line. Select all that apply. ​

Answers

One point that would lie on the second line is (0,-4). Another  possible point on the 2nd line is (0, 12).

To find the equation of the first line, we can use the slope-intercept form:

y = mx + b

where m is the slope and b is the y-intercept. The slope of the line passing through (-3,8) and (1,9) can be found using the formula:

m = (y2 - y1) / (x2 - x1)

m = (9 - 8) / (1 - (-3))

m = 1/4

Using one of the points and the slope, we can find the y-intercept:

8 = (1/4)(-3) + b

b = 9

So the equation of the first line is:

y = (1/4)x + 9

To find the equation of the second line, we need to use the fact that it is perpendicular to the first line. The slopes of perpendicular lines are negative reciprocals, so the slope of the second line is:

m2 = -1/m1 = -1/(1/4) = -4

Using the point-slope form, we can write the equation of the second line:

y - 4 = -4(x + 2)

y - 4 = -4x - 8

y = -4x - 4

To find a point that lies on this line, we can plug in a value for x and solve for y. For example, if we let x = 0, then:

y = -4(0) - 4

y = -4

So the point (0,-4) lies on the second line.

Therefore, another point that would lie on the second line is (0,-4).

More on points: https://brainly.com/question/29395724

#SPJ11

What is the product of the expression, 5x(x2)? (1 point)
25x2
10x
5x3
5x2

Answers

The product of the expression 5x(x²) is 5x³.


1. Write down the given expression: 5x(x²)
2. Apply the distributive property, which states that a(b + c) = ab + ac. In this case, we have a single term inside the parentheses, so the expression becomes: 5x * x²
3. Multiply the coefficients (numbers) together: 5 * 1 = 5
4. Multiply the variables together, which means adding the exponents since they have the same base (x): x¹* x² = x⁽¹⁺²⁾ = x³
5. Combine the result from steps 3 and 4: 5x³

The product of the expression 5x(x²) can be found by multiplying the coefficients (numbers) and adding the exponents of the variables (letters). In this case, we have 5 times x times x squared.

5 times x equals 5x, and x squared means x times x, so we can rewrite the expression as:

5x(x²) = 5x(x*x) = 5x³

So, the product of the expression 5x(x²) is 5x³.

Know more about expression here:

https://brainly.com/question/14083225

#SPJ11

Quadrilateral FGHJ was dilated with the origin as the center of dilation to create quadrilateral F' G′ H′ J′.



Which rule best represents the dilation that was applied to quadrilateral FGHJ to create quadrilateral F' G′ H′ J′?



A. (x, y) à (5/7x, 5/7y)


B. (x, y) à (1. 4x , 1. 4y)


C. (x, y) à (x + 1, y + 2)


D. (x, y) à (x - 2, y + 1)









Which rule best represents the dilation that was applied to quadrilateral FGHJ to create quadrilateral F' G′ H′ J′?

Answers

The rule that best represents the dilation that was applied to quadrilateral FGHJ to create quadrilateral F'G'H'J' is option B, which is (x, y) à (1.4x, 1.4y).

What is the dilation rule used to create quadrilateral F'G'H'J' from FGHJ?

A dilation is a transformation that changes the size of an object without changing its shape. It is performed by multiplying the coordinates of each point by a scale factor.

In this case, the center of dilation is the origin, which means that the coordinates of each point are multiplied by the same scale factor in both the x and y directions.

The scale factor can be found by comparing the corresponding side lengths of the two quadrilaterals. In this case, the scale factor is 1.4, which means that the lengths of the sides of F'G'H'J' are 1.4 times the lengths of the corresponding sides of FGHJ.

Learn more about dilation

brainly.com/question/13176891

#SPJ11

find the extremes of 4x−4y subject to condition x2 + 2y2 = 1

Answers

To find the extremes of 4x−4y subject to the condition x2 + 2y2 = 1, we can use the method of Lagrange multipliers.

First, we set up the Lagrange equation:

∇f(x,y) = λ∇g(x,y)

where f(x,y) = 4x-4y and g(x,y) = x2 + 2y2 - 1.

Taking partial derivatives, we have:

∂f/∂x = 4
∂f/∂y = -4
∂g/∂x = 2x
∂g/∂y = 4y

Setting these equal to their respective Lagrange multipliers, we have:

4 = 2λx
-4 = 4λy
x2 + 2y2 = 1

Solving for x and y in terms of λ, we get:

x = 2λ/4 = λ/2
y = -λ/4

Substituting these back into the constraint equation, we have:

(λ/2)2 + 2(-λ/4)2 = 1
λ2/4 + λ2/8 = 1
3λ2/8 = 1
λ2 = 8/3

Taking the positive and negative square roots of λ2, we have:

λ = ±2√2/3

Substituting these values back into x and y, we get:

For λ = 2√2/3:
x = (2√2/3)/2 = √2/3
y = -(2√2/3)/4 = -√2/6

For λ = -2√2/3:
x = (-2√2/3)/2 = -√2/3
y = -(-2√2/3)/4 = √2/6

Now we can find the extreme values of f(x,y) by plugging in these values of x and y:

f(√2/3, -√2/6) = 4(√2/3) - 4(-√2/6) = 4√2
f(-√2/3, √2/6) = 4(-√2/3) - 4(√2/6) = -4√2

Therefore, the maximum value of 4x-4y subject to the condition x2 + 2y2 = 1 is 4√2 and the minimum value is -4√2.

To learn more about derivative  visit;

brainly.com/question/30365299

#SPJ11

A new sign is being designed for the cityâs skate park. Knowing the exact angles is necessary for fitting the sign where it will hang. The architect started to write in the angles, but went home sick before she could finish. It is up to you to fill in the missing angles. For 4 of the 8 missing angles, explain your answer

Answers

Using trigonometry, we can solve for the missing angles to find them as 18.43°, 45°, 45°, and 18.43°.

The sign is mounted on a sloped surface, which means that we'll need to use some trigonometry to find the missing angles.

Let's concentrate on the sign's upper right corner, where the letters x and y are absent from two perspectives. The magnitude of angle x may be determined using trigonometry.

Let's begin by sketching a right triangle that has an angle x. The triangle's two sides may be represented by the sign's vertical and horizontal lines, with the addition of a third side to join the top right corner of the sign to the sloping area below.

Since the sign is an octagon, we know that each interior angle is 135°. Therefore, the measure of angle y must be:

y = 180 - 135 = 45°

Now, let's look at the right triangle that includes angle x. We know that the hypotenuse of the triangle is the sloped surface of the sign, which has a length of 4.5 meters. We also know that the opposite side of the triangle is the height of the sign above the ground, which has a length of 1.5 meters.

Using trigonometry, we can find the measure of angle x by taking the inverse tangent of the opposite side over the adjacent side:

tan(x) = opposite/adjacent = 1.5/4.5 = 1/3

x = tan⁻¹(1/3) ≈ 18.43°

Therefore, the measure of angle x is approximately 18.43 degrees.

Hence, using trigonometry, we can solve for the missing angles to find them as 18.43°, 45°, 45°, and 18.43°.

To learn more about trigonometry, refer to:

https://brainly.in/question/2685053

#SPJ4

Find the solution and also verify your answer , under root 12 x 12 x - 4 is equals under root 4 x + 8

Answers

The solution to the given equation is x = -3/4 or x = 1.

What values of x satisfy the equation √(12x² - 4) = √(4x + 8)?

In order to find the solution, we start by squaring both sides of the equation to eliminate the square roots:

12x² - 4 = 4x + 8

Next, we simplify the equation by moving all the terms to one side:

12x² - 4x - 12 = 0

Now we can factor the quadratic equation:

4x² - x - 3 = 0

By factoring or using the quadratic formula, we find that the equation can be written as:

(4x + 3)(x - 1) = 0

Setting each factor equal to zero gives us the solutions:

4x + 3 = 0 or x - 1 = 0

Solving for x in each equation yields:

x = -3/4 or x = 1

Therefore, the solution to the given equation is x = -3/4 or x = 1.

Learn more about square roots

brainly.com/question/29286039

#SPJ11

20 pt if someone can answer this!!!
Pls help.

The median value is____

Answers

Answer:

The median value is 45.

Step-by-step explanation:

"The median is the middle number in a sorted, ascending or descending list of numbers"

The middle number here is 45

50

Step-by-step explanation:

I put the explanation on the attachment. please see it.

Part A: Sydney made $18. 50 selling lemonade, by the cup, at her yard sale. She sold each cup for $0. 50 and received a $3 tip from a neighbor. Write an equation to represent this situation. (4 points)



Part B: Daria made a profit of $21. 00 selling lemonade. She sold her lemonade for $0. 75 per cup, received a tip of $3 from a neighbor, but also had to buy each plastic cup she used for $0. 10 per cup. Write an equation to represent this situation. (4 points)



Part C: Explain how the equations from Part A and Part B differ. (2 points)

Answers

Part A: The equation to represent this situation is: 0.50x + 3 = 18.50
Part B: The equation to represent this situation is: 0.75x + 3 - 0.10x = 21.00
Part C: The equations differ in the following ways:
1. Sydney's equation involves only the price per cup and the tip, while Daria's equation also considers the cost of the plastic cups.
2. The price per cup for Sydney and Daria are different.

Part A: The equation to represent this situation is:

18.50 = 0.50x + 3

Where x represents the number of cups of lemonade sold.

Part B: The equation to represent this situation is:

21.00 = 0.75x + 3 - 0.10x

Where x represents the number of cups of lemonade sold.

Part C: The equations from Part A and Part B differ in that Part B takes into account the cost of each plastic cup used to serve the lemonade, while Part A only considers the revenue from selling each cup of lemonade and the tip received. This means that the profit in Part B is calculated after deducting the cost of each plastic cup from the revenue earned, while the profit in Part A does not account for any costs incurred.

To know more about equation, refer to the link below:

https://brainly.com/question/24870633#

#SPJ11

Other Questions
4.1 Name and Describe FIVE conditions that led to the formation of cyclone Freddy What is the answer? I don't understand. Mark's aunt is researching the effect of car pollution on smog levels in southern California. She wants to learn more about this. Which keyword(s) should she type into an Internet search engine? A. aunt B. car research C. southern California levels D. car pollution in California A political candidate feels that she performed particularly well in the most recent debate against her opponent. Her campaign manager polled a random sample of 400 likely voters before the debate and a random sample of 500 likel voters after the debate. The 95% confidence interval for the true difference (post-debate minus pre-debate) in proportions of likely voters who would vote for this candidate was (-0. 014, 0. 064). What was the difference (pre- debate minus post-debate) in the sample proportions of likely voters who said they will vote for this candidate? According to kotter, leadership is primarily about coping with what?. Your assignment, you need to start at President Harry Truman and go through President Ronald Reagan and examine how they handled the Cold War during their time in office. You need to do a paragraph per President. Include how they dealt with the Cold War during their time, the years they were in office, and any pertinent data/info in regard to their leadership during that time. Complete thoughts, complete sentences, complete paragraphs! One of the unintended consequences of prohibition was that it served _________________________. An angle measure 94 less than the measure of its supplementary angle. What is the measure of each angle? Stade avogadro's hypothesis what are its applications, prove that hydrogen hydrogen and oxygen gases Which of the following statements is not true? a. stress can impact hunger and eating behaviors. b. humans have an innate preference for spicy and sour foods. c. parental behaviors and preferences influence eating behaviors in children. d. social situations can influence eating behaviors. please select the best answer from the choices provided a b c d An art studio offers classes for painting and pottery. Each painting class is 1hour long. Each pottery class is 1. 5 hours long. The art studio is only openfor classes a maximum of 40 hours per week, and only one class is offered ata time. Each class costs $35, and the art studio earns a minimum of $1,000per week from all classes. Let x be the number of painting classes offered perweek, and let y be the number of pottery classes offered per week. The critical point for water lies at 275 C and 3.2 atm, calculate the DHvap of water. Public domainUse the graph and your knowledge of U.S. history to answer the question. Which of these explains the differences in Union vs. Confederate firearm production during the Civil War? (5 points) HELP!!! You have two substances, both of which have the same boiling point (or attraction between their molecules).The first substance is made from molecules that are small (just a few atoms bonded together), and the second substance is made from mol that are larger (many atoms bonded together).How can it be possible for two such different molecules to yield substances with the same boiling point? Describe the kinds of intermolecu attractions that must be involved and any other properties of the molecules that could cause this result. PLEASE HELPConstruct the indicated confidence interval for the population mean u using the t-distribution. Assume the population is normally distributed.C=0.90, x=13.7, s =3.0, n= 10 Why do expanding, aging stars become cooler and more luminous? an overproduction of energy causes the outer layers of gas to expand, whereby the energy is absorbed and the temperature increases. the resulting increase in radiated energy leads to increased luminosity. a decrease in energy causes the outer layers of gas to expand and the temperature to decrease. the resulting increase in radiated energy leads to increased luminosity. a decrease in energy causes the outer layers of gas to expand and the temperature to decrease, and the resulting increase in surface area leads to increased luminosity. an overproduction of energy causes the outer layers of gas to expand. when this happens, the surface area increases rapidly while the temperature decreases rather slowly, and therefore the luminosity increases. relate exports of raw material from farming and fishery to global economy A rectangle has vertices located at the points: A(-1 1/4,3 1/2), B(2 2/3,3 1/2), C(2 2/3,-1 3/4), D(-1 1/4,-1 3/4). Find the length of BC Mike has some candies. he gave some to his friend. then, his mom gave him twice as much as he had in the beginning. how much did he have in the beginning if he has a total of 60 candies now? Mongol warriors have four horses each. they can eat and sleep on horseback. no army willtravel so far and so fast until